(20 intermediate revisions by 2 users not shown)
Line 1: Line 1:
= [[ECE PhD Qualifying Exams|ECE Ph.D. Qualifying Exam]] in "Communication, Networks, Signal, and Image Processing" (CS)  =
+
[[Category:ECE]]
 +
[[Category:QE]]
 +
[[Category:CNSIP]]
 +
[[Category:problem solving]]
 +
[[Category:image processing]]
  
= [[ECE-QE_CS5-2011|Question 5, August 2011]], Part 1 =
+
<center>
 +
<font size= 4>
 +
[[ECE_PhD_Qualifying_Exams|ECE Ph.D. Qualifying Exam]]
 +
</font size>
  
:[[ECE-QE_CS5-2011_solusion-1|Part 1]],[[ECE-QE CS5-2011 solusion-2|2]]]
+
<font size= 4>
 +
Communication, Networking, Signal and Image Processing (CS)
  
 +
Question 5: Image Processing
 +
</font size>
 +
 +
August 2011
 +
</center>
 +
----
 +
----
 +
=Part 1 =
 +
Jump to [[ECE-QE_CS5-2011_solusion-1|Part 1]],[[ECE-QE CS5-2011 solusion-2|2]]
 
----
 
----
  
Line 41: Line 58:
  
 
===== <math>\color{blue}\text{Solution 1:}</math>  =====
 
===== <math>\color{blue}\text{Solution 1:}</math>  =====
 +
<math>\color{green}
 +
\text{Recall should be added:}
 +
</math>
 +
 +
<math>\color{green}
 +
f(am,bn) \overset{DTFT}{\Leftrightarrow } \frac{1}{|a||b|}F(\frac{\mu}{|a|},\frac{\nu}{|b|})
 +
</math>
 +
 +
<math>\color{green}
 +
sinc(m,n) \overset{DTFT}{\Leftrightarrow } rect(\mu,\nu)
 +
</math>
  
 
<math>
 
<math>
Line 53: Line 81:
 
sinc(m,n) \rightarrow rect(\mu)rect(\nu)
 
sinc(m,n) \rightarrow rect(\mu)rect(\nu)
 
</math>
 
</math>
 +
  
 
<math>
 
<math>
Line 58: Line 87:
 
</math>
 
</math>
  
<math>
+
 
H(e^{j\mu},e^{j\nu})
+
<font face="serif"><span style="font-size: 19px;"><math>
 +
= H(e^{j\mu},e^{j\nu})
 +
</math></span></font>
 +
 
 +
<math>\color{green}
 +
\text{Here, the student uses the Separability property of the sinc and rect functions.}
 
</math>
 
</math>
 
----
 
----
  
<math>\color{blue}\text{b) Sketch the frequency response for } |\mu| < 2\pi \text{ and } |nu| < 2\pi \text{ when } T = \frac{1}{2}  
+
<math>\color{blue}\text{b) Sketch the frequency response for } |\mu| < 2\pi \text{ and } |\nu| < 2\pi \text{ when } T = \frac{1}{2}  
 
</math><br>  
 
</math><br>  
  
[[Image:.jpg]]
+
<math>\color{green}
 +
\text{Recall should be added:}
 +
</math>
 +
 
 +
<math>\color{green}
 +
rect(t) = \left\{\begin{matrix}
 +
1, for |t|\leq \frac{1}{2}
 +
\\
 +
0, otherwise
 +
\end{matrix}\right.
 +
</math>
 +
 
 +
 
 +
<math>{\color{green}
 +
\text{Here, the following descriptions should be clarified:}
 +
}</math>
 +
 
 +
<font face="serif"><span style="font-size: 19px;"><math>{\color{green}
 +
\text{Using the separability property for rect function, for } T = \frac{1}{2} { we have:}
 +
}</math></span></font>
 +
 
 +
<font face="serif"><span style="font-size: 19px;"><math>{\color{green}
 +
H(e^{j\mu},e^{j\nu}) = \frac{1}{T^2} rect(\frac{\mu}{T},\frac{\nu}{T})
 +
}</math></span></font>
 +
 
 +
<font face="serif"><span style="font-size: 19px;"><math>{\color{green}
 +
= 4 rect(2\mu)rect(2\nu)
 +
}</math></span></font>
 +
 
 +
 
 +
[[Image:QE_11_CS5_1_b.png]]
  
 +
<font face="serif"><span style="font-size: 19px;"><math>{\color{red}
 +
\text{In this sketch it is not mentioned that the gain is } 4.
 +
}</math></span></font>
 
----
 
----
  
Line 73: Line 140:
  
 
<math>
 
<math>
T = \frac{1}{2}, H(e^{j\mu},e^{j\nu}) = 4rect(2\mu)rect(\nu)
+
T = \frac{1}{2}, H(e^{j\mu},e^{j\nu}) = 4rect(2\mu)rect(2\nu)
 
</math>
 
</math>
  
[[Image:.jpg]]
+
[[Image:QE_11_CS5_1_b_sol2.PNG]]
  
  
Line 84: Line 151:
  
 
<math>\color{blue}\text{Solution 1:}</math>  
 
<math>\color{blue}\text{Solution 1:}</math>  
 +
 +
<math>\color{green}
 +
\text{Recall should be added:}
 +
</math>
 +
 +
<math>\color{green}
 +
f \left ( A \begin{bmatrix}
 +
m
 +
\\
 +
n
 +
\end{bmatrix} \right) \overset{DTFT}{\Leftrightarrow } \frac{1}{|A|^{-1}}F([\mu, \nu] A^{-1})
 +
</math>
 +
 +
 +
<font color="#ff0000"><span style="font-size: 17px;">'''<font face="serif"></font><math>\color{green}
 +
\text{ In this case, A}= \begin{bmatrix}
 +
\frac{1}{\sqrt{2}} &\frac{1}{\sqrt{2}} \\
 +
-\frac{1}{\sqrt{2}} &\frac{1}{\sqrt{2}}
 +
\end{bmatrix} \text{, hence:}
 +
</math>'''</span></font><font color="#ff0000"><span style="font-size: 17px;">
 +
</span></font>
  
  
Line 99: Line 187:
 
\frac{1}{\sqrt{2}} &\frac{1}{\sqrt{2}} \\  
 
\frac{1}{\sqrt{2}} &\frac{1}{\sqrt{2}} \\  
 
-\frac{1}{\sqrt{2}} &\frac{1}{\sqrt{2}}  
 
-\frac{1}{\sqrt{2}} &\frac{1}{\sqrt{2}}  
\end{bmatrix} = \begin{pmatrix}
+
\end{bmatrix} \cdot \begin{pmatrix}
 
mT\\  
 
mT\\  
 
nT
 
nT
Line 109: Line 197:
  
 
<math>
 
<math>
\text{As } |A| = 1, A^{-1} = A^T,  
+
\text{As } |A| = 1, A^{-1} = A^T, sinc \left( A \begin{pmatrix}
 +
mT\\
 +
nT
 +
\end{pmatrix} \right)
 +
 
 +
\overset{\mathcal{F}}{\rightarrow} F \left( A \begin{pmatrix}
 +
\mu\\
 +
\nu
 +
\end{pmatrix} \right)
 +
 
 +
 
 
</math>
 
</math>
  
Line 118: Line 216:
 
----
 
----
  
<math>\color{blue}\text{d) Sketch the frequency response for } |\mu| < 2\pi \text{ and } |nu| < 2\pi \text{ when } T = \frac{1}{2}  
+
<math>\color{blue}\text{d) Sketch the frequency response for } |\mu| < 2\pi \text{ and } |\nu| < 2\pi \text{ when } T = \frac{1}{2}  
 
</math><br>  
 
</math><br>  
  
 
<math>\color{blue}\text{Solution 1:}</math>  
 
<math>\color{blue}\text{Solution 1:}</math>  
  
[[Image:.jpg]]
+
<math>\color{green}
 +
\text{Recall should be added: Since A is an orthogonal matrix, this transformation is rotationally invariant.}
 +
</math>
 +
 
 +
<font face="serif"><span style="font-size: 19px;"><math>\color{green}
 +
H(e^{j\mu},e^{j\nu}) = \frac{1}{T^2} rect \left ( \frac{(\mu + \nu)}{\sqrt{2}T},\frac{(\nu - \mu)}{\sqrt{2}T} \right )
 +
</math></span></font>
 +
 
 +
<font face="serif"><span style="font-size: 19px;"><math>\color{green}
 +
= 4 rect \left (\sqrt{2} (\mu + \nu),\sqrt{2}(\nu - \mu) \right )
 +
</math></span></font>
 +
 
 +
<font face="serif"><span style="font-size: 19px;"><math>\color{green}
 +
\text{Or}
 +
</math></span></font>
 +
 
 +
<font face="serif"><span style="font-size: 19px;"><math>\color{green}
 +
= 4 rect \left (\sqrt{2} (\mu + \nu) \right) rect \left (\sqrt{2}(\nu - \mu) \right )
 +
</math></span></font>
 +
 
 +
 
 +
[[Image:QE_11_CS5_1_d.PNG]]
 +
 
 +
 
 +
<font face="serif"><span style="font-size: 19px;"><math>{ \color{red}
 +
\text{This sketch is partially correct: The cut-offs should be divided by } 4!
 +
}</math></span></font>
 +
 
 +
<font face="serif"><span style="font-size: 19px;"><math>{ \color{red}
 +
\text{ Also, it should be mentioned that the gain is} 4.
 +
}</math></span></font>
  
 
----
 
----
Line 129: Line 257:
 
<math>\color{blue}\text{Solution 2:}</math><br>  
 
<math>\color{blue}\text{Solution 2:}</math><br>  
  
sol2 here
+
<math>
 +
T = \frac{1}{2}, H(e^{j\mu},e^{j\nu}) = 4rect(\sqrt{2}(\mu + \nu))rect(\sqrt{2}(\nu - \mu))
 +
</math>
 +
 
 +
[[Image:QE_11_CS5_1_d_sol2.PNG]]
 +
 
 
----
 
----
 
<math>\color{blue}\text{e) Calculate } y(m,n) \text{ when } x(m,n)=1.</math><br>
 
<math>\color{blue}\text{e) Calculate } y(m,n) \text{ when } x(m,n)=1.</math><br>
Line 153: Line 286:
 
<math>\color{blue}\text{Solution 2:}</math><br>  
 
<math>\color{blue}\text{Solution 2:}</math><br>  
  
sol2 here
+
<math>
 +
y(m,n) = x(m,n) \cdot H(e^{j0},e^{j0}) = 4
 +
</math>
 +
 
 +
<math>\color{red}
 +
\text{The final answer is correct, but the student has skipped some parts of the derivation and the notations do not sound right.}
 +
</math>
 +
 
 
----
 
----
  
Line 165: Line 305:
 
----
 
----
  
[[ECE PhD Qualifying Exams|Back to ECE Qualifying Exams (QE) page]]
+
[[ECE PhD Qualifying Exams|Back to ECE Qualifying Exams (QE) page]]
 
+
[[Category:ECE]] [[Category:QE]] [[Category:CS]] [[Category:Problem_solving]]
+

Latest revision as of 10:31, 13 September 2013


ECE Ph.D. Qualifying Exam

Communication, Networking, Signal and Image Processing (CS)

Question 5: Image Processing

August 2011



Part 1

Jump to Part 1,2


 $ \color{blue}\text{Consider the following discrete space system with input } x(m,n) \text{ and output } y(m,n). $

                $ \color{blue} y(m,n) = \sum_{k=-\infty}^{\infty}{\sum_{l=-\infty}^{\infty}{x(m-k,n-l)h(k,l)}}. $


$ \color{blue} \text{For parts a) and b) let} $
                $ \color{blue} h(m,n)=sinc(mT,nT) $
$ \color{blue} \text{where } T\leq1. $


$ \color{blue} \text{For parts c), d), and e) let} $
                $ \color{blue} h(m,n)=sinc\left( \frac{(n+m)T}{\sqrt[]{2}},\frac{(n-m)T}{\sqrt[]{2}} \right) $
$ \color{blue} \text{where } T\leq1. $


$ \color{blue}\text{a) Calculate the frequency response, }H \left( e^{j\mu},e^{j\nu} \right). $

$ \color{blue}\text{Solution 1:} $

$ \color{green} \text{Recall should be added:} $

$ \color{green} f(am,bn) \overset{DTFT}{\Leftrightarrow } \frac{1}{|a||b|}F(\frac{\mu}{|a|},\frac{\nu}{|b|}) $

$ \color{green} sinc(m,n) \overset{DTFT}{\Leftrightarrow } rect(\mu,\nu) $

$ H(e^{j\mu},e^{j\nu}) = \frac{1}{T^2} rect(\frac{\mu}{T},\frac{\nu}{T}) $


$ \color{blue}\text{Solution 2:} $

$ sinc(m,n) \rightarrow rect(\mu)rect(\nu) $


$ \Rightarrow sinc(mT,nT) \rightarrow \frac{1}{T^2}rect(\frac{\mu}{T})rect(\frac{\nu}{T}) $


$ = H(e^{j\mu},e^{j\nu}) $

$ \color{green} \text{Here, the student uses the Separability property of the sinc and rect functions.} $


$ \color{blue}\text{b) Sketch the frequency response for } |\mu| < 2\pi \text{ and } |\nu| < 2\pi \text{ when } T = \frac{1}{2} $

$ \color{green} \text{Recall should be added:} $

$ \color{green} rect(t) = \left\{\begin{matrix} 1, for |t|\leq \frac{1}{2} \\ 0, otherwise \end{matrix}\right. $


$ {\color{green} \text{Here, the following descriptions should be clarified:} } $

$ {\color{green} \text{Using the separability property for rect function, for } T = \frac{1}{2} { we have:} } $

$ {\color{green} H(e^{j\mu},e^{j\nu}) = \frac{1}{T^2} rect(\frac{\mu}{T},\frac{\nu}{T}) } $

$ {\color{green} = 4 rect(2\mu)rect(2\nu) } $


QE 11 CS5 1 b.png

$ {\color{red} \text{In this sketch it is not mentioned that the gain is } 4. } $


$ \color{blue}\text{Solution 2:} $

$ T = \frac{1}{2}, H(e^{j\mu},e^{j\nu}) = 4rect(2\mu)rect(2\nu) $

QE 11 CS5 1 b sol2.PNG



$ \color{blue}\text{c) Calculate the frequency response, }H \left( e^{j\mu},e^{j\nu} \right). $

$ \color{blue}\text{Solution 1:} $

$ \color{green} \text{Recall should be added:} $

$ \color{green} f \left ( A \begin{bmatrix} m \\ n \end{bmatrix} \right) \overset{DTFT}{\Leftrightarrow } \frac{1}{|A|^{-1}}F([\mu, \nu] A^{-1}) $


$ \color{green} \text{ In this case, A}= \begin{bmatrix} \frac{1}{\sqrt{2}} &\frac{1}{\sqrt{2}} \\ -\frac{1}{\sqrt{2}} &\frac{1}{\sqrt{2}} \end{bmatrix} \text{, hence:} $


$ H(e^{j\mu},e^{j\nu}) = \frac{1}{T^2} rect \left ( \frac{(\mu + \nu)}{\sqrt{2}T},\frac{(\nu - \mu)}{\sqrt{2}T} \right ) $


$ \color{blue}\text{Solution 2:} $

$ \left ( \frac{(n + m)T}{\sqrt{2}},\frac{(n - m)T}{\sqrt{2}} \right) = \begin{bmatrix} \frac{1}{\sqrt{2}} &\frac{1}{\sqrt{2}} \\ -\frac{1}{\sqrt{2}} &\frac{1}{\sqrt{2}} \end{bmatrix} \cdot \begin{pmatrix} mT\\ nT \end{pmatrix} = A \cdot \begin{pmatrix} mT\\ nT \end{pmatrix} $

$ \text{As } |A| = 1, A^{-1} = A^T, sinc \left( A \begin{pmatrix} mT\\ nT \end{pmatrix} \right) \overset{\mathcal{F}}{\rightarrow} F \left( A \begin{pmatrix} \mu\\ \nu \end{pmatrix} \right) $

$ = \frac{1}{T^2} rect \left ( \frac{(\mu + \nu)}{\sqrt{2}T},\frac{(\nu - \mu)}{\sqrt{2}T} \right ) $


$ \color{blue}\text{d) Sketch the frequency response for } |\mu| < 2\pi \text{ and } |\nu| < 2\pi \text{ when } T = \frac{1}{2} $

$ \color{blue}\text{Solution 1:} $

$ \color{green} \text{Recall should be added: Since A is an orthogonal matrix, this transformation is rotationally invariant.} $

$ \color{green} H(e^{j\mu},e^{j\nu}) = \frac{1}{T^2} rect \left ( \frac{(\mu + \nu)}{\sqrt{2}T},\frac{(\nu - \mu)}{\sqrt{2}T} \right ) $

$ \color{green} = 4 rect \left (\sqrt{2} (\mu + \nu),\sqrt{2}(\nu - \mu) \right ) $

$ \color{green} \text{Or} $

$ \color{green} = 4 rect \left (\sqrt{2} (\mu + \nu) \right) rect \left (\sqrt{2}(\nu - \mu) \right ) $


QE 11 CS5 1 d.PNG


$ { \color{red} \text{This sketch is partially correct: The cut-offs should be divided by } 4! } $

$ { \color{red} \text{ Also, it should be mentioned that the gain is} 4. } $


$ \color{blue}\text{Solution 2:} $

$ T = \frac{1}{2}, H(e^{j\mu},e^{j\nu}) = 4rect(\sqrt{2}(\mu + \nu))rect(\sqrt{2}(\nu - \mu)) $

QE 11 CS5 1 d sol2.PNG


$ \color{blue}\text{e) Calculate } y(m,n) \text{ when } x(m,n)=1. $


$ \color{blue}\text{Solution 1:} $


$ Y(e^{j\mu},e^{j\nu}) = \delta(e^{j\mu},e^{j\nu}) \cdot H(e^{j\mu},e^{j\nu}) $

$ = \frac{1}{T^2} rect (0,0) = 4 $

$ \Rightarrow y(m,n) = 4\delta(m,n) $


$ \color{blue}\text{Solution 2:} $

$ y(m,n) = x(m,n) \cdot H(e^{j0},e^{j0}) = 4 $

$ \color{red} \text{The final answer is correct, but the student has skipped some parts of the derivation and the notations do not sound right.} $


"Communication, Networks, Signal, and Image Processing" (CS)- Question 5, August 2011

Go to


Back to ECE Qualifying Exams (QE) page

Alumni Liaison

Ph.D. 2007, working on developing cool imaging technologies for digital cameras, camera phones, and video surveillance cameras.

Buyue Zhang